Stay ahead of learning milestones! Enroll in a class over the summer!

G
Topic
First Poster
Last Poster
No topics here!
Putnam 2015 B4
Kent Merryfield   21
N Sep 20, 2024 by Aiden-1089
Let $T$ be the set of all triples $(a,b,c)$ of positive integers for which there exist triangles with side lengths $a,b,c.$ Express \[\sum_{(a,b,c)\in T}\frac{2^a}{3^b5^c}\]as a rational number in lowest terms.
21 replies
Kent Merryfield
Dec 6, 2015
Aiden-1089
Sep 20, 2024
Putnam 2015 B4
G H J
The post below has been deleted. Click to close.
This post has been deleted. Click here to see post.
Kent Merryfield
18574 posts
#1 • 5 Y
Y by mathematicsy, Zfn.nom-_nom, Adventure10, Mango247, cubres
Let $T$ be the set of all triples $(a,b,c)$ of positive integers for which there exist triangles with side lengths $a,b,c.$ Express \[\sum_{(a,b,c)\in T}\frac{2^a}{3^b5^c}\]as a rational number in lowest terms.
Z K Y
The post below has been deleted. Click to close.
This post has been deleted. Click here to see post.
BOGTRO
5818 posts
#2 • 3 Y
Y by Adventure10, Mango247, cubres
Sol
Better finish

EDIT: Grading-related question: in contest my solution was essentially identical to the above, except at some point I bungled the computation and ended up with an extra factor of $\frac{5}{13}$ from somewhere (so final answer of $\frac{85}{273}$). Is that more of a -1 or a -9 level mistake?
This post has been edited 2 times. Last edited by BOGTRO, Dec 6, 2015, 10:33 PM
Reason: grading question
Z K Y
The post below has been deleted. Click to close.
This post has been deleted. Click here to see post.
Kent Merryfield
18574 posts
#3 • 3 Y
Y by Adventure10, Mango247, cubres
I get that the sum is $\frac{1292}{6643},$ noting that $6643=7\cdot 13\cdot 73.$ But I don't really trust my arithmetic.
Z K Y
The post below has been deleted. Click to close.
This post has been deleted. Click here to see post.
v_Enhance
6871 posts
#4 • 14 Y
Y by rkm0959, nahiphog, math998, FS123, e_plus_pi, XbenX, math31415926535, mathematicsy, cttg8217, kamatadu, Adventure10, Mango247, NicoN9, cubres
By the Ravi substitution,

\begin{align*}
	\sum_{(a,b,c) \in T} \frac{2^a}{3^b5^c}
	&= \sum_{x,y,z \ge 1 \text{ odd}}
	\frac{2^\frac{y+z}{2}}{3^{\frac{z+x}{2}} 5^{\frac{x+y}{2}}}
	+ \sum_{x,y,z \ge 2 \text{ even}}
	\frac{2^\frac{y+z}{2}}{3^{\frac{z+x}{2}} 5^{\frac{x+y}{2}}} \\
	&= \sum_{u,v,w \ge 1} \frac{2^{v+w}}{3^{w+u}5^{u+v}}
	\left( 1 + \frac{2^{-1}}{3^{-1}5^{-1}} \right) \\
	&= \frac{17}{2} \sum_{u,v,w \ge 1}
	\left(\frac{1}{15}\right)^u \left(\frac25\right)^v \left(\frac23\right)^w \\
	&= \frac{17}{2} \frac{\frac{1}{15}}{1-\frac{1}{15}}
	\frac{\frac25}{1-\frac25}\frac{\frac23}{1-\frac23} \\
	&= \frac{17}{21}.
\end{align*}
No idea why this was B4; it was by far the easiest problem on the test for me. I knew how to do it in less than a second on seeing it and the rest was just arithmetic. (We use the Ravi substitution all the time on olympiad inequalities, so it's practically a reflex for me.)
Z K Y
The post below has been deleted. Click to close.
This post has been deleted. Click here to see post.
pi37
2079 posts
#5 • 4 Y
Y by v_Enhance, Adventure10, Mango247, cubres
I did Ravi solution for the my writup, but as a question, how carefully would one have to deal with convergence issues here? I think everything is fine with the Ravi approach because our three numbers are all then less than $1$.

Here's another approach: let $x=2,y=\frac{1}{3},z=\frac{1}{5}$, and consider the formal power series
\[
f(x,y,z)=\sum_{(a,b,c)\in T} x^ay^bz^c
\](so we're basically throwing convergence issues out the window.) Let $S_a$ be the set of positive triples with $a\ge b+c$, and define $S_b,S_c$ similarly. Then
\begin{align*}
f&=\sum_{a,b,c\ge 1} x^ay^bz^c-\sum_{cyc}\sum_{(a,b,c)\in S_a} x^ay^bz^c\\
&=\frac{xyz}{(1-x)(1-y)(1-z)}-\sum_{cyc}\sum_{b,c\ge 1} \frac{x(xy)(xz)}{(1-x)(1-xy)(1-xz)}\\
&=\frac{xyz+x^2y^2z^2}{(1-xy)(1-yz)(1-zx)}
\end{align*}where the last equality follows after sufficient expansion. Plugging in $x,y,z$ gives us the answer.
Z K Y
The post below has been deleted. Click to close.
This post has been deleted. Click here to see post.
mathocean97
606 posts
#6 • 3 Y
Y by Adventure10, Mango247, cubres
You don't even need Ravi substitution. I wrote this up in 10 minutes just by doing casework on $b = c$, $b - c = x > 0, c - b = x > 0.$ Everything works out absurdly nicely.
Z K Y
The post below has been deleted. Click to close.
This post has been deleted. Click here to see post.
mlerma
3 posts
#7 • 3 Y
Y by Adventure10, Mango247, cubres
Do we need to be concerned about convergence here? All terms are positive and we can rearrange them in any way, change the order of summation (Tonelli's theorem), etc., without modifying the sum, whether it is finite or infinite. If at the end we get a positive number less than infinity that means that the series converges. An example in which the sum is not going to converge is
\[\sum_{(a,b,c)\in T} \frac{3^a}{2^b 5^c}\]If we repeat the computations we will get that the intermediate sum $\sum_{w=1}^{\infty} \frac{3^w}{2^w}$ diverges, and so the sum is going to be infinite.
Z K Y
The post below has been deleted. Click to close.
This post has been deleted. Click here to see post.
jmerry
12096 posts
#8 • 4 Y
Y by kamatadu, Adventure10, Mango247, cubres
The grubby way, for the sake of completionism:

The first two sides of the triangle $a,b$ can be any positive integers. The third side $c$ then runs between $|a-b|+1$ and $a+b-1$. This lets us write the sum
$S=\sum_{a=1}^{\infty}\sum_{b=1}^{\infty}\sum_{c=|a-b|+1}^{a+b-1}2^{a}3^{-b}5^{-c}$.
To deal with that absolute value, we split into one half with $a\le b$ and another with $a>b$:
$S=\sum_{a=1}^{\infty}2^{a}\sum_{b=1}^{a}3^{-b}\sum_{c=a-b+1}^{a+b-1}5^{-c}+\sum_{a=1}^{\infty}2^{a}\sum_{b=a+1}^{\infty}3^{-b}\sum_{c=b-a+1}^{a+b-1}5^{-c}$
From here, it's all arithmetic, evaluating finite and infinite geometric series.
$S=\sum_{a=1}^{\infty}2^{a}\sum_{b=1}^{a}3^{-b}\sum_{c=a-b+1}^{a+b-1}5^{-c}+\sum_{a=1}^{\infty}2^{a}\sum_{b=a+1}^{\infty}3^{-b}\sum_{c=b-a+1}^{a+b-1}5^{-c}$
$S=\sum_{a=1}^{\infty}2^{a}\sum_{b=1}^{a}3^{-b}\frac{5^{-a+b-1}-5^{-a-b}}{1-5^{-1}}+\sum_{a=1}^{\infty}2^{a}\sum_{b=a+1}^{\infty}3^{-b}\frac{5^{-b+a-1}-5^{-a-b}}{1-5^{-1}}$
$S=\sum_{a=1}^{\infty}2^{a}\sum_{b=1}^{a}3^{-b}\left(\frac14 5^{-a+b}-\frac54 5^{-a-b}\right)+\sum_{a=1}^{\infty}2^{a}\sum_{b=a+1}^{\infty}3^{-b}\left(\frac14 5^{-b+a}-\frac54 5^{-a-b}\right)$
$S=\frac14\sum_{a=1}^{\infty}\left(\frac25\right)^{a}\sum_{b=1}^{a}\left(\frac53\right)^{b}-\frac54\sum_{a=1}^{\infty}\left(\frac25\right)^{a}\sum_{b=1}^{a}\left(\frac1{15}\right)^{b}+\frac14\sum_{a=1}^{\infty}10^{a}\sum_{b=a+1}^{\infty}\left(\frac1{15}\right)^{b}-\frac54\sum_{a=1}^{\infty}\left(\frac25\right)^{a}\sum_{b=a+1}^{\infty}\left(\frac1{15}\right)^{b}$
The second and fourth terms are sums of the same quantity with complementary indices. Merge them:
$S=\frac14\sum_{a=1}^{\infty}\left(\frac25\right)^{a}\sum_{b=1}^{a}\left(\frac53\right)^{b}+\frac14\sum_{a=1}^{\infty}10^{a}\sum_{b=a+1}^{\infty}\left(\frac1{15}\right)^{b}-\frac54\sum_{a=1}^{\infty}\left(\frac25\right)^{a}\sum_{b=1}^{\infty}\left(\frac1{15}\right)^{b}$
$S=\frac14\sum_{a=1}^{\infty}\left(\frac25\right)^{a}\frac{\frac53-\left(\frac53\right)^{a+1}}{1-\frac53}+\frac14\sum_{a=1}^{\infty}10^{a}\frac{\left(\frac1{15}\right)^{a+1}}{1-\frac1{15}}-\frac54\sum_{a=1}^{\infty}\left(\frac25\right)^{a}\frac{\frac1{15}}{1-\frac1{15}}$
$S=\frac58\sum_{a=1}^{\infty}\left(\frac23\right)^{a}-\frac58\sum_{a=1}^{\infty}\left(\frac25\right)^{a}+\frac1{56}\sum_{a=1}^{\infty}\left(\frac{10}{15}\right)^a-\frac5{56}\sum_{a=1}^{\infty}\left(\frac25\right)^{a}$
$S=\frac58\cdot 2-\frac58\cdot\frac23+\frac1{56}\cdot 2-\frac5{56}\cdot \frac23$

$S=\frac54-\frac5{12}+\frac1{28}-\frac{5}{84}=\frac{105}{84}-\frac{35}{84}+\frac{3}{84}-\frac{5}{84}=\frac{68}{84}=\frac{17}{21}$

OK, I chickened out a little on the grubbiness. My scratchwork left the $b>a$ side for later, thus not seeing the two sums that can be merged. That's where factors of $73$ in the denominator come in; evaluating those sums without the merge leads to geometric series with ratio $\frac{2}{75}$. A $13$ in the denominator indicates a geometric series with ratio $\frac2{15}$ somewhere, and that must be a mistake.
Z K Y
The post below has been deleted. Click to close.
This post has been deleted. Click here to see post.
xuanji
1 post
#9 • 3 Y
Y by Adventure10, Mango247, cubres
Can you cite Ravi's substitution without proof?
Z K Y
The post below has been deleted. Click to close.
This post has been deleted. Click here to see post.
champion999
1530 posts
#10 • 6 Y
Y by mutasimmim12, gausskarl, Adventure10, Mango247, exopeng, cubres
It's a substitution, how would you even "prove a substitution"?
Z K Y
The post below has been deleted. Click to close.
This post has been deleted. Click here to see post.
Grizzy
920 posts
#11 • 1 Y
Y by cubres
We use a Ravi substitution. Let $a=\frac{x+y}2$ and cyclic permutations, for positive integers $x,y,z$. We note that $x, y, z$ have the same parity.

Let $S_1$ be the sum

\[\sum_{x,y,z}\frac{2^{\frac{x+y}2}}{3^{\frac{y+z}2}5^{\frac{z+x}2}}\]
where $x,y,z$ are odd and $S_2$ be the same sum when $x,y,z$ are even. We compute

\[S_1= \left(\sum_{x\text{ odd}} \sqrt{\frac25}^x\right)\left(\sum_{y\text{ odd}} \sqrt{\frac23}^y\right)\left(\sum_{z\text{ odd}} \sqrt{\frac1{15}}^z\right)=\left(\frac{\sqrt{\frac25}}{1-\frac25}\right)\left(\frac{\sqrt{\frac23}}{1-\frac23}\right)\left(\frac{\sqrt{\frac1{15}}}{1-\frac1{15}}\right)=\frac57\]
and

\[S_2=\left(\sum_{x\text{ even}} \sqrt{\frac25}^x\right)\left(\sum_{y\text{ even}} \sqrt{\frac23}^y\right)\left(\sum_{z\text{ even}} \sqrt{\frac1{15}}^z\right)=\left(\frac{\frac25}{1-\frac25}\right)\left(\frac{\frac23}{1-\frac23}\right)\left(\frac{\frac1{15}}{1-\frac1{15}}\right)=\frac2{21}\]
so our desired answer is

\[S_1 + S_2 = \boxed{\frac{17}{21}}.\]
Z K Y
The post below has been deleted. Click to close.
This post has been deleted. Click here to see post.
brainiacmaniac31
2170 posts
#12 • 1 Y
Y by cubres
Solution
Z K Y
The post below has been deleted. Click to close.
This post has been deleted. Click here to see post.
bobthegod78
2982 posts
#13 • 1 Y
Y by cubres
Dang i got this wrong the first time because I thought $2^1=1$ :clown:

let's break this into cases and not care about $a$ for now, and we will sum it over the possible values of $a$.

Case 1: $b>c$. Using the necessary triangle inequality conditions and letting $b=c+d$, we get $$\displaystyle \sum_{c=1}^{\infty} \sum_{d=1}^\infty \sum_{a=d+1}^{2c+d-1} \frac{2^a}{3^{c+d} 5^c}.$$First let's factor out the denominator and simplify a bit as it only depends on $c,d$ which are defined before the third summation. We get $$\displaystyle \sum_{c=1}^{\infty} \sum_{d=1}^\infty \frac{1}{15^c 3^d}\sum_{a=d+1}^{2c+d-1} 2^a.$$Now notice how the inner sum is just a geometric series. We can sum it using the finite geometric series formula. The number of terms is $2c+d-1-d-1+1=2c-1$. The common ratio is $2$. So we can sum it to get $2^{d+1}(2^{2c-1}-1)$. Plug this back into the summation to get $$\displaystyle \sum_{c=1}^{\infty} \sum_{d=1}^\infty \frac{2^{d+1}(2^{2c-1}-1)}{15^c 3^d}.$$Again, we factor out stuff to make it easier and we get $$\displaystyle \sum_{c=1}^{\infty} \frac{2^{2c-1}-1}{15^c} \sum_{d=1}^\infty \frac{2^{d+1}}{3^d}.$$Now this is just an infinite geometric series with first term $\frac 43$ and common ratio $\frac 23$, which has sum $4$. We can factor this out as its just a constant. So our sum is currently $$4\displaystyle \sum_{c=1}^{\infty} \frac{2^{2c-1}-1}{15^c}.$$Now this is a simple sum which we can break apart to solve. Doing that, we get $$\displaystyle 4\sum_{c=1}^{\infty} \frac{2^{2c-1}}{15^c} - 4\sum_{c=1}^\infty \frac{1}{15^c}.$$The first sum is an infinite geo series with first term $\frac{2}{15}$ and common ratio $\frac{4}{15}$, so it has sum $\frac{2}{11}$. The second sums to $\frac{1}{14}$. Now multiplying and adding, we get $\frac{8}{11} - \frac{2}{7} = \frac{34}{77}.$

Case 2: $c>b$. Using the necessary triangle inequality conditions and letting $c=b+d$, we get $$\displaystyle \sum_{b=1}^{\infty} \sum_{d=1}^\infty \sum_{a=d+1}^{2b+d-1} \frac{2^a}{3^b 5^{b+d}}.$$First let's factor out the denominator and simplify a bit as it only depends on $b,d$ which are defined before the third summation. We get $$\displaystyle \sum_{b=1}^{\infty} \sum_{d=1}^\infty \frac{1}{15^b 5^d}\sum_{a=d+1}^{2b+d-1} 2^a.$$Now notice how the inner sum is just a geometric series. We can sum it using the finite geometric series formula. The number of terms is $2b+d-1-d-1+1=2b-1$. The common ratio is $2$. So we can sum it to get $2^{d+1}(2^{2b-1}-1)$. Plug this back into the summation to get $$\displaystyle \sum_{c=1}^{\infty} \sum_{d=1}^\infty \frac{2^{d+1}(2^{2b-1}-1)}{15^b 5^d}.$$Again, we factor out stuff to make it easier and we get $$\displaystyle \sum_{b=1}^{\infty} \frac{2^{2b-1}-1}{15^b} \sum_{d=1}^\infty \frac{2^{d+1}}{5^d}.$$Now this is just an infinite geometric series with first term $\frac 45$ and common ratio $\frac 25$, which has sum $\frac 43$. We can factor this out as its just a constant. So our sum is currently $$\frac 43\displaystyle \sum_{b=1}^{\infty} \frac{2^{2b-1}-1}{15^b}.$$Now this is a simple sum which we can break apart to solve. Doing that, we get $$\displaystyle \frac 43\sum_{b=1}^{\infty} \frac{2^{2b-1}}{15^b} - \frac 43\sum_{b=1}^\infty \frac{1}{15^b}.$$The first sum is an infinite geo series with first term $\frac{2}{15}$ and common ratio $\frac{4}{15}$, so it has sum $\frac{2}{11}$. The second sums to $\frac{1}{14}$. Now multiplying and adding, we get $\frac{8}{33} - \frac{2}{21} = \frac{34}{231}.$

Case 3: $b=c$. Now the easy thing about this case is that there is no minimum. But there is still a maximum for $a$. We can also just consider one variable $x$ which is $b$ and $c$ both. So our sum is a simple $$\displaystyle \sum_{x=1}^\infty \sum_{a=1}^{2x-1} \frac{2^a}{15^x}.$$Using a similar method, this reduces to $$\displaystyle \sum_{x=1}^\infty \frac{1}{15^x}\sum_{a=1}^{2x-1} 2^a,$$which by finite geo series is $$\displaystyle \sum_{x=1}^\infty \frac{2^{2x}-2}{15^x}.$$We break this apart to get $$\displaystyle \sum_{x=1}^\infty \frac{2^{2x}}{15^x} - \sum_{x=1}^\infty \frac{2}{15^x}.$$Using the infinite geometric series for each sum, we get $\frac{4}{11} - \frac{1}{7} = \frac{17}{77}.$

If we sum all the cases, we get $$\frac{187}{231} = \boxed{\frac{17}{21}}$$.
This post has been edited 1 time. Last edited by bobthegod78, Oct 5, 2021, 12:31 AM
Z K Y
The post below has been deleted. Click to close.
This post has been deleted. Click here to see post.
HamstPan38825
8857 posts
#15 • 1 Y
Y by cubres
Ravi substitution: either $x, y, z$ are all integers or they are rational numbers with denominator 2 in lowest terms. For the first case, for instance, we just want $$\sum_{y \geq 1}\sum_{z\geq 1}\sum_{x \geq 1} \frac{2^{\frac{y+z}2}}{3^{\frac{x+y}2}5^{\frac{x+z}2}}.$$Evaluate the sum layer by layer for both cases to get $$\frac{\sqrt{\frac 23} \cdot \sqrt{\frac 25} \cdot \sqrt{\frac 1{15}} + \frac 23 \cdot \frac 25 \cdot \frac 1{15}}{\left(1-\frac 23\right)\left(1-\frac 25\right)\left(1-\frac 1{15}\right)} = \frac{17}{21}$$is the answer.
Z K Y
The post below has been deleted. Click to close.
This post has been deleted. Click here to see post.
metricpaper
54 posts
#16 • 1 Y
Y by cubres
Given $b$ and $c$, the triple $(a,b,c)$ is in $T$ if and only if $a$ satisfies $|b-c|<a<b+c$ (triangle inequality). So we can rewrite the sum as
\[S=\sum_{(a,b,c)\in T}\frac{2^a}{3^b5^c}=\sum_{b\geq 1}\sum_{c\geq 1}\left(\frac{1}{3^b 5^c} \sum_{\substack{a \leq b+c-1 \\ a\geq |b-c|+1}} 2^a\right)=\sum_{b\geq 1}\sum_{c\geq 1}\left(\frac{1}{3^b 5^c}\left(2^{b+c}-2^{|b-c|+1}\right)\right).\]We can split this up:
\[S=\sum_{b\geq 1}\sum_{c\geq 1}\left(\frac23\right)^b\left(\frac25\right)^c-\sum_{b\geq 1}\sum_{c\geq 1}\frac{2^{|b-c|+1}}{3^b5^c}.\]The first term on the RHS can be written as $\Sigma_{b\geq 1}(\tfrac23)^b\cdot \Sigma_{c\geq 1}(\tfrac25)^c=\tfrac43$.

To get rid of the absolute values on the second term, we split it up again based on whether $c\geq b$ or $c<b$:
\[\sum_{b\geq 1}\sum_{c\geq 1}\frac{2^{|b-c|+1}}{3^b5^c}=-2\sum_{b\geq 1}\left(3^{-b}\left(\sum_{c\geq b}\frac{2^{c-b}}{5^c}+\sum_{c=1}^{b-1}\frac{2^{b-c}}{5^c}\right)\right).\]If we set $k=c-b$, then $\Sigma_{c\geq b}\tfrac{2^{c-b}}{5^c}=\Sigma_{k\geq 0}\tfrac{2^k}{5^{k+b}}=\tfrac{1}{3\cdot 5^{b-1}}$. Similarly, the second term $\Sigma_{c=1}^{b-1}\tfrac{2^{b-c}}{5^c}$ is a geometric series that evaluates to $\tfrac{2\cdot 10^{b-1}-2}{9\cdot 5^{b-1}}$. Substituting these both in, we are left with a summation in just one variable $b$. Evaluating the resulting combination of geometric series, we find that it is $-\tfrac{11}{21}$.

So $S=\tfrac{4}{3}-\tfrac{11}{21}=\boxed{\tfrac{17}{21}}$.
Z K Y
The post below has been deleted. Click to close.
This post has been deleted. Click here to see post.
Spectator
657 posts
#17 • 1 Y
Y by cubres
We apply the Ravi substitution. Then, we get
\[\displaystyle\sum_{x, y, z}{\frac{2^{x+y}}{3^{y+z}5^{x+z}}} = \displaystyle\sum_{x,y,z}{\biggl(\frac{2}{5}\biggr)^{x}\biggl(\frac{2}{3}\biggr)^{y}\biggl(\frac{1}{15}\biggr)^{z}}\]We have that $x,y,z\in \mathbb{Z}^{+}$ or $x, y, z \in \mathbb{Z}+\frac{1}{2}$. If $x,y,z\in \mathbb{Z}^{+}$, then we have
\[\frac{\frac{2}{5}\cdot\frac{2}{3}\cdot\frac{1}{15}}{(1-\frac{2}{5})(1-\frac{2}{3})(1-\frac{1}{15})} = \frac{2}{21}\]If $x, y, z \in \mathbb{Z}+\frac{1}{2}$, then we have
\[\frac{\sqrt{\frac{4}{225}}}{(1-\frac{2}{5})(1-\frac{2}{3})(1-\frac{1}{15})} = \frac{5}{7}\]Adding these up, we get $\boxed{\frac{17}{21}}$.
Z K Y
The post below has been deleted. Click to close.
This post has been deleted. Click here to see post.
Etkan
1551 posts
#18 • 2 Y
Y by NicoN9, cubres
champion999 wrote:
It's a substitution, how would you even "prove a substitution"?

It means to prove that you can actually do the substitution. Ravi doesn't work if the numbers are not the sides of a triangle, in the same way that you can't say "let $x=t^2$ for $t\in \mathbb{R}$" if $x<0$.
Now I never did the Putnam (I'm not from the US/Canada), so take this with a grain of salt, but I'm pretty sure that Ravi is well known enough that you can just cite it without proof.
Z K Y
The post below has been deleted. Click to close.
This post has been deleted. Click here to see post.
exopeng
21 posts
#19 • 1 Y
Y by cubres
Reflecting back, I feel this problem is somewhat ambiguous in that it doesn't explicitly discount degenerate triangles.
Z K Y
The post below has been deleted. Click to close.
This post has been deleted. Click here to see post.
chakrabortyahan
377 posts
#20 • 1 Y
Y by cubres
Cute problem!! Somewhat easy with ravi's substitution (though thinking of the substitution was not easy for me so at first was doing some nasty calculations)
$\blacksquare\smiley$
@below umm...not entirely for the convergence I guess ...like the three variables $p,q,r$ are not dependent on each other..so we can split them like we do for finite sums...it is a different point that the splitting is valid as every infinite GP converges...
Attachments:
This post has been edited 1 time. Last edited by chakrabortyahan, Apr 29, 2024, 10:30 AM
Z K Y
The post below has been deleted. Click to close.
This post has been deleted. Click here to see post.
Sreemani76
90 posts
#21 • 1 Y
Y by cubres
Why are we able to separate the triple summation into 3 separate GPs? Is it because all the 3 GPs converge individually?
This post has been edited 1 time. Last edited by Sreemani76, Apr 29, 2024, 9:24 AM
Z K Y
The post below has been deleted. Click to close.
This post has been deleted. Click here to see post.
sanyalarnab
929 posts
#23 • 1 Y
Y by cubres
Surely this is a mistake (credits:chakrabortyahan)
Attachments:
Z K Y
The post below has been deleted. Click to close.
This post has been deleted. Click here to see post.
Aiden-1089
277 posts
#24 • 1 Y
Y by cubres
Substitute positive integers $x,y,z$, where $x=b+c-a, y=a+c-b, z=a+b-c$, such that $a=\frac{y+z}{2}, b=\frac{x+z}{2}, c=\frac{x+y}{2}$. Note that $x,y,z$ are either all odd or all even.
$\sum_{(a,b,c)\in T}\frac{2^a}{3^b5^c} = \sum \frac{2^\frac{y+z}{2}}{3^\frac{x+z}{2}5^\frac{x+y}{2}} = \sum \left( \sqrt{\frac{1}{15}} \right)^x \left( \sqrt{\frac{2}{5}} \right)^y  \left( \sqrt{\frac{2}{3}} \right)^z$
$= \left( 1+ \sqrt{\frac{1}{15}} \cdot \sqrt{\frac{2}{5}} \cdot \sqrt{\frac{2}{3}} \right) \cdot \frac{\sqrt{\frac{1}{15}}}{1-\frac{1}{15}} \cdot \frac{\sqrt{\frac{2}{5}}}{1-\frac{2}{5}} \cdot \frac{\sqrt{\frac{2}{3}}}{1-\frac{2}{3}} = \frac{17}{15} \cdot \frac{2}{15} \cdot \frac{15}{14} \cdot \frac{5}{3} \cdot 3 = \frac{17}{21}$
Z K Y
N Quick Reply
G
H
=
a